Download as pdf or txt
Download as pdf or txt
You are on page 1of 32

Study Materials

• JEE Main & Advanced – Free Study Material

• NEET UG – Free Study Material

• NCERT Solutions for Class 1 to 12

• NCERT Books PDF for Class 1 to 12

• ICSE & ISC Free Study Material

• Free Study Material for Kids Learning (Grade 1 to 5)

• Olympiad Free Study Material

• Reference Books (RS Aggarwal, RD Sharma, HC Verma, Lakhmir


Singh, Exemplar and More)

• Previous Year Question Paper CBSE & State Boards

• Sample Papers

• Access All Free Study Material Here

Vedantu Innovations Pvt. Ltd.


Score high with a personal teacher, Learn LIVE Online!
www.vedantu.com

Study Material
Downloaded from Vedantu

FREE LIVE ONLINE

MASTER CLASSES
FREE Webinars by Expert Teachers

About Vedantu
Vedantu is India’s largest LIVE online teaching platform
with best teachers from across the country.

Vedantu offers Live Interactive Classes for JEE, NEET, KVPY,


NTSE, Olympiads, CBSE, ICSE, IGCSE, IB & State Boards for
Students Studying in 6-12th Grades and Droppers. Register for FREE

Awesome Master Teachers


Anand Prakash Pulkit Jain Vamsi Krishna
B.Tech, IIT Roorkee B.Tech, IIT Roorkee B.Tech, IIT Bombay
Co-Founder, Vedantu Co-Founder, Vedantu Co-Founder, Vedantu

My mentor is approachable and guides me My son loves the sessions and I can
in my future aspirations as well. already see the change.

Student - Ayushi Parent - Sreelatha

10,04,600+ 9,49,900+ 95% 95% Students of Regular

Hours of LIVE Learning Happy Students  Top Results


Tuitions on Vedantu scored
above 90% in exams!

FREE MASTER CLASS SERIES


 For Grades 6-12th targeting JEE, CBSE, ICSE & much more Register for FREE
 Free 60 Minutes Live Interactive classes everyday
 Learn from the Master Teachers - India’s best Limited Seats!
CLASS 7
PAPER 1
SECTION 01 LOGICAL REASONING
Directions( 1-2) . Four of the following five are alike in a
certain way and so form a group. Which is the one that does not
belong to that group?
1. (A) CBF (B) EBJ (C) KBV (D) DET
Ans. [D]
C B F
3 × 2 6
E B J
5 × 2 10
K B V
11 ×2 22
D E T
4 × 5 20

2. (A) EY (B) CI (C) DP (D) LO


Ans. [D]
Except [D], second letter is square of the first letter of all other
options

Directions (3-4): Each of the following questions consists of two


sets of figures. Figures 1, 2, 3 and 4 constitute the problem set
while figures (a), (b), (c) and (d) constitute the answer set. There
is a definite relationship between figures (1) and (2) a similar
relationship between figures (3) and (4) by selecting a suitable
figure from the answer set that would replace the problem mark
(?) in fig (4).

?
3. (1) (2) (3) (4)

(a) (b) (c) (d)

(A) a (B) b (C) c (D) d


Ans. [A]

?
4. (1) (2) (3) (4)

(a) (b) (c) (d)

(A) a (B) b (C) c (D) d


Ans. [B]

5. In the following question, select the number which can


Beplaced at the sign of question mark (?) from the give-
n alternatives.

(A) 20
(B) 21
(C) 22
(D) 24
Ans. [C]
3*4+6=18
2*8+5=21
3*7+1=22

6. From a circular sheet of paper with a radius 20 cm, four circles


of radius 5 cm each are cut out. What is the ratio of the uncut to
the cut portion?
(A) 1: 3 (B) 4: 1
(C) 3: 1 (D) 4: 3
Ans. [C]
Area of the original paper = (20)2 = 400cm2. The total cut
portion area = 4()(5)2 = 100cm2. Therefore, area of the
uncut (shaded) portion = (400 – 100) = 300cm2.
Hence, the required ratio = 300: 100= 3: 1.
7. The cost of diamond varies directly as the square of its weight.
Once, this diamond broke into four pieces with weights in the
ratio 1: 2: 3: 4. When the pieces were sold, the merchant got
Rs. 70,000 less. Find the original price of the diamond.
(A) Rs. 1.4 lakh
(B) Rs. 2 lakh
(C) Rs. 1 lakh
(D) Rs. 2.1 lakh
Ans. [C]
Let the original weight of the diamond be 10x. Hence, its
original price will be k (100x2) . . . where k is a constant.
The weights of the pieces after breaking are x, 2x, 3x and 4x.
Therefore, their prices will be kx2, 4kx2, 9kx2 and 16kx2. So
the total price of the pieces = (1 + 4 + 9 + 16) kx2 = 30kx2.
Hence, the difference in the price of the original diamond and
its pieces = 100kx2 – 30kx2 = 70kx2 = 70000.
Hence, kx2 = 1000 and the original price = 100kx2 = 100 ×
1000 = 100000 = Rs. 1 lakh.

8. Find the value of


1 3

1 4
1 3
4 1
3 3
1 1
2 2
1 2
3
2

13 15
(A) 7
(B) 7
11 17
(C) 21
(D) 28

Ans. [B]

9. In a watch, the minute hand crosses the hour hand for the third
time exactly after every 3 hr 18 min and 15 s of watch time. What
is the time gained or lost by this watch in one day?
(A) 14 min 10 s lost
(B) 13 min 50 s lost
(C) 13 min 20 s gained
(D) 14 min 40 s gained
Ans. [B]
In a watch that is running correct, the minute hand should
5
cross the hour hand once in every 65  11 min. So they should
 720  2060
ideally cross three times once in 3   11 min = 196.36
 11 
min. But in the watch under consideration they meet after
every 3 hr, 18 min and 15 s, i.e. (3 × 60 + 18 + 15
60

793
4
min =
198.25 min. In other words, our watch is actually losing time
(as it is slower than the normal watch). Hence, when our
watch elapsed 198.25 min, it actually should have elapsed
196.36 min. So in a day, when our watch will elapse (60 × 24)
= 1440, it should actually elapse 1440  196.36 
198.25 
= 1426.27.
 

Hence, the amount of time lost by our watch in one day =


(1440 – 1426.27) = 13.73, i.e. 13 min and 50 s
(approximately).
10. I sold two watches for Rs. 300 each, one at the loss of 10%
and the other at the profit of 10%. What is the percentage of
loss(–) or profit(+) that resulted from the transaction?
(A) (+)10
(B) (–)1
(C) (+)1
(D) (–)10
Ans. [B]
In this case, we need not use the data that SP = Rs. 300 each.
This has to be used only to figure out that the SP of both the
articles is the same. Also since the profit percentage on one is
equal to the loss percentage on the other, viz. 10% effectively,
2
it will be a loss given by (10)
100
 1% . Hence, the correct answer

is (–)1.

11. Samir was standing facing East. He turned to his right and
walked 5 metres, again turned to his right and walked 7 metres.
Then he turned to his left and walked 4 metres. Which direction
is he facing now?
(A) North
(B) South
(C) West
(D) North-West
Ans. [A]
Movement of Samir is shown below :
Hence in the end be will be facing to South.

12. How many such pairs of letters are there in the word
‘EXPERIMENT’, each of which has as many letters between
them in the word as they have in the English alphabet?
(A) None
(B) One
(C) Two
(D) Three

Ans. [C]

13. In a row of children facing North, Shweta is fifteenth from


the left and Jyoti is third to the left of Shweta. Ram who is
seventh to the right of Jyoti is fifth from the right end of the
row. What is Shweta’s position from the right end?
(A) 12th
(B) 10th
(C) 8th
(D) 9th
Ans. [D]
Explanation:

Shweta is on ninth position from the left end.

Directions (Q.14-15): These questions are based on the


following information—
‘P @ Q’ means ‘P is mother of Q’.
‘P $ Q’ means ‘P is husband of Q’.
‘P # Q’ means ‘P is sister of Q’.
‘P * Q’ means ‘P is son of Q’.
14. If A#J*T$R@L, then which of the following is definitely
true?
(A) L is brother of F
(B) F is sister of L
(C) F is brother of J
(D) L is brother of J
Ans. [B]
Explanation:
F # J → F is the sister of J, J * T → J is the son of T T $ R
→ T is the husband of R and R @ L → R is the mother of L.
Hence F is sister of L.

15. Which of the following indicates the relationship ‘R is


daughter of T’?
(A) R#F*B@T
(B) R#F*B$T
(C) T@B#R*F
(D) T@B#F*R
Ans. [B]
Explanation:
R # F → R is the sister of F, F * B → F is the son of B B $
T → B is the husband of T.
∴ R # F * B $ T → R is the daughter of T.

SECTION 02 Mathematical Reasoning


16. If ba  13 , bc  2 , dc  21 , ed  3 and ef  14 , then what is the value of abc
def

?
3 27
(A) 8
(B) 8

3 27
(C) 4
(D) 4

Ans. [A]
a 1 b 2
   a:b : c  2 : 6 :3
b 3 c 1

Similarly a : b : c : d : e : f = 6 : 18 : 9 : 18 : 6 : 24
abc 6  18  9 3
  
def 18  6  24 8

Hence, option (A) is the correct answer.

n
17. Consider a sequence where the nth term, tn 
(n  2)
,n  1,2,...

The value of t3  t 4  t5  ....  t53 equals:


2
(A) 495

2
(B) 477
12
(C) 55

1
(D) 1485

Ans. [A]
t3  t 4  t5  ....  t53
3 4 5 51 52 53
    ....   
5 6 7 53 54 55
34 2
 
54  55 495

Hence, option (A) is the correct answer.

18. The length, breadth and height of a room are in the ratio 3 : 2
: 1. If the breadth and height are halved while the length is
doubled, then the total area of the four walls of the room will
(A) remain the same
(B) decrease by 13.64%
(C) decrease by 15%
(D) decrease by 30%
Ans. [D]
Let the initial length, breadth and height of the room be
3x, 2x and x respectively.
Initial area of the four walls = 2 ( 3x + 2x) x = 10x2
The new dimensions are: length = 6x, breadth = x and height
x
 .
2
x
New area of the four walls = 2(6x + x) 2
 7x2

Therefore, percentage decrease


10x2  7x2
  100  30%
10x2

Ans. [D]

19. The sum of four consecutive two-digit odd numbers, when


divided by 10, becomes a perfect square.
Which of the following can possibly be one of these four
numbers?
(A) 21 (B) 25
(C) 41 (D) 67
Ans. [C]
By option (3), if four consecutive odd numbers are 37, 39, 41
and 43, then sum of these 4 numbers is 160.
When divided by 10, we get 16, which is a perfect square.
41 is one of the odd numbers.

20. What are the values of x and y that satisfy both the equations?
8 3
20.7x  31.25y 
27
1
40.3x  90.2y  8.(81)5

(A) x = 2,y = 5
(B) x = 2.5, y = 6
(C) x = 3, y = 5
(D) x = 5, y = 2
Ans. [D]
Equation (ii) can be written as
40.3x× 90.2y= 8 × (81)1/5
⇒(22)0.3x(32)0.2y= 8 ×(81)1/5
⇒20.6x30.4y= 23 ×(34)1/5= 23 ×34/5
⇒0.6x = 3 ⇒x = 5 and 0.4y = 4/5
⇒y =2
If we put the values of x and y in first equation these values
satisfy the first equation also.
So the answer is x = 5, y = 2

21. An equilateral triangle BPC is drawn inside a square ABCD.


What is the value of the angle APD in degrees?
(A) 75 (B) 90
(C) 120 (D) 150
Ans. [D]

∠PBC = ∠CPB = ∠BPC = 60° (L's of equilateral triangle)


PC = CD (both a)
180o  30o
Also CPD  PDC 
2
 75o

Similarly, ∠BAP = ∠BPA = 75°


APD = 360° − 75° − 75° − 60° = 150°

22. How many numbers between 1 to 1000 (both excluded) are


both squares and cubes?
(A) none (B) 1
(C) 2 (D) 3
Ans. [B]
Try with whole cubes as they are fewer in number 43 = 64 and
82 = 64.

23. The sum of the numbers from 1 to 100, which are not divisible
by 3 and 5.
(A) 2946 (B) 2732
(C) 2632 (D) 2317
Ans. [D]
Sum of all numbers,
100
S [1  100]  5050 .... using AP
2

Similarly, sum of multiples of 3, Required sum


33 = 5050-1683-1050
S3  [3  99]  1683
2
= 2117
Similarly, sum of multiples of 5,
20
S5  [5  100]  1050
2
24. For what value of x does 10x • 1002x = 10005?
(A) 1 (B) 2
(C) 3 (D) 4
Ans. [C]
We can rewrite
10x • 1002x = 10005 as 105x = 1015
10x • 1002x = 10x • (102)2x
10x • 104x = (103)5
105x = 1015
Since the bases are equal, we can set the exponents equal,
giving us 5x = 15, Solving the equation gives us x = 3

25. Aakash, Rahul, and Sunil went on a vacation and agreed to


split the costs evenly, During their trip Aakash paid ` 105
Rahul paid ` 175. in order to share costs equally, Aakash gave
Sunil t rupees, and Rahul gave Sunil d rupees. What is t – d?

(A) 15 (B) 20
(C) 30 (D) 35
Ans. [B]
The total amount paid is 105 + 125 + 175 = 405. To get how
much each should have paid, we do 405/3 = 135
Thus, we know that Aakash needs to give Sunil 30 rupees, and
Rahul 10 rupees. This means that t - d = 30 = 20.
22014 + 22012
26. What is the value of 22014 - 22012
?
(A) – 1 (B) 1
5
(C) 3
(D) 2013
Ans. [C]
Factoring out, we get 
22012 22  1 .
2 2012
2
2
 1

Cancelling out the 22012 from the numerator and denominator,


we see that it simplifies to 53 .

27. A flower bouquet contains pink roses, red roses, pink


carnations, and red carnations. One third of the pink flowers are
roses, three fourths of the red flowers are carnations, and six
tenths of the flowers are pink. What percent of the flowers are
carnations?
(A 15 (B) 30
(C) 60 (D) 70
Ans. [D]
Let the total amount of flowers be x. Thus, the number of pink
flowers is 0.6x, and the number of red flowers is 0.4x. The
number of pink carnations is 23 0.6x   0.4x and the number red
carnation is
3
4
0.4x   0.3x
. Summing these, the total number of carnations is
0.4x + 0.3x = 0.7x. Dividing, we see that 0.7x
x
 0.7  70%
28. Which of the following is equal to 9- 6 2 + 9+6 2 ?

(A) 3 2 (B) 2 6

(C) 3 3 (D) 6
Ans. [B]
We find the answer by squaring, then square rooting the
expression.
96 2  96 2

 
2
 96 2  96 2

 96 2 2 9  6 2 9  6 2   9  6 2

 18  2 9  6 2 9  6 2 
 
2
 18  2 92  6 2

 18  2 81  72

 18  2 9

 18  6

 24  2 6

29. In 1991 the population of a town was a perfect square. Ten


years later, after an increase of 150 people, the population was
9 more than a perfect square. Now, in 2011, with an increase
another 150 people, the population is once again a perfect
square. Which of the following is closest to the percent
growth of the town’s population during this twenty-year
period?
(A) 42 (B) 47
(C) 57 (D) 62
Ans. [D]

30. Let n (>1) be a composite integer such that n is not an integer.


Consider the n following statements:
A: n has a perfect integer-valued divisor which is greater than
1 and less than n
B: n has a perfect integer-valued divisor which is greater than
n but less than n

(A) Both A and B are false


(B) A is true but B is false
(C) A is false but B is true
(D)Both A and B are true
Ans. [D]
Let n = 6.
Therefore, n  6  2.4
Therefore, divisors of 6 are 1, 2 and 3.
If we take 2 as divisor, then, n >2 >1.
Therefore, statement A is true.
If we take 3 as divisor, then 6 > 3 > 2.4, i.e., n > 3 > n .
Therefore, statement B is also true.

31. Let a, b, c, d and e be integers such that a = 6b = 12c, and 2b


= 9d = 12 e. Then which of the following pairs contains a
number that is not an integer?
 a b
(A)  27 , e 
 

 a c
(B)  36 , e 
 

 a bd 
(C)  12 , 18 
 

a c
(D) 6 , d
 

Ans. [D]
a = 6b = 12c and 2b = 9d = 12e.
Dividing the first equations by 12 and second by 36,
we get

32. The equation 7x–1 + 11x–1 = 170 has


(A) No solution (B) One solution
(C) Two solutions (D) Three solutions
Ans. [B]
7x-1 + 11x-1 = 170
We can see that the RHS is a multiple of 10
11x-1 has 1 in its units place.
 7x – 1 should have 9 in its units place.
The lowest value for which this is true is x=3
72 + 112 = 170
We can see that the for any other value of x, which is greater
than 3,
7x–1 + 11x-1> 170

33. In 2011, Plasma – a pharmaceutical company – allocated Rs.


4.5 × 107 for Research and Development. In 2012, the
company allocated Rs, 60,000,000 for Research and
Development. If each year the funds are evenly divided
among 2×102 departments, how much more will each
department receive this year than it did last year?
(A) Rs. 2.0 × 105 (B) Rs. 7.5 × 105
(C) Rs. 7.5 × 104 (D) Rs. 2.5 × 107
Ans. [C]
Funds allocated for Research & Development in 2011
= Rs. 4.5 × 107
Funds allocated for Research & Development in 2012 = Rs.
6 × 107
Difference in the funds = Rs. 1.5 × 107
Difference in the funds received by each Department
1.5  107
  7.5  104
2  102
1.5  107
  7.5  104
2  102
34. In a circular field, there is a rectangular tank of length 130
m and breadth 110 m. If the area of the land portion of the
field is 20350 m2 then the radius of the field is
(A) 85 m (B) 95 m
(C) 105 m (D) 115 m
Ans. [C]
Area of a circular field = r2
Area of tank = 130 × 110 = 14300 m2
∴r2 – 14300 = 20350
∴r2 = 34650
r2 = 1575 × 7 = 225 × 49
r = 15 × 7 = 105 m

35. It takes 15 seconds for a train travelling at 60 km/hour to cross


entirely another train half its length and travelling in opposite
direction at 48 km/hour. It also passes a bridge in 51 seconds.
The length of the bridge is
(A) 550 m (B) 450 m
(C) 500 m (D) 600 m
Ans. [A]

SECTION 03 EVERYDAY MATHEMATICS


36. A student gets an aggregate of 60% marks in five subjects in
the ratio 10: 9: 8: 7: 6. If the passing marks are 50% of the
maximum marks and each subject has the same maximum
marks, in how many subjects did he pass the examination?
(A) 2 (B) 3
(C) 4 (D) 5
Ans. [C]
Let his marks be 100, 90, 80, 70 and 60 in the five subjects.
Hence, totally he has scored 400 marks. This constitutes only 60%
of the total marks. Hence, total marks  400 0.6
 667 Since the total

marks in each subject is the same, hence maximum marks in each


subject will be  667 
5 
133 Out of this 50% is the passing marks. In

other words, to pass in a subject he needs to score 66.5 marks. We


can see that only in one subject he scored less than this, viz. 60.
Hence, he passed in 4 subjects.

37. After allowing a discount of 11.11%, a trader still makes a


gain of 14.28%. At how many percentage above the cost price
does he mark on his goods?
(A) 28.56% (B) 35%
(C) 22.22% (D) None of these
Ans. [A]
38. A dealer buys dry fruits at Rs.100, Rs.80 andRs. 60 per
kilogram. He mixes them in the ratio 3: 4: 5 by weight, and
sells at a profit of 50%. At what price per kilogram does he
sell the dry fruit?
(A) Rs.80 (B) Rs.100
(C) Rs.95 (D) None of these
Ans. [D]
Let he mix 3 kg, 4 kg and 5 kg of dry fruits at Rs. 100, Rs. 80 and
at Rs.60 per kilogram respectively. Hence, his effective cost of
the dry fruits per kilogram should be the weighted average

In order to make a 50% profit, he will have to sell it at

per kilogram. Since none of the answer-choices confirms this, the


answer is (D).

39. An express train travelling at 80 km/hr overtakes a goods


train, twice as long and going at 40 km/hr on a parallel track,
in 54 s. How long will the express train take to cross a
platform of 400 m long?
(A) 36 s (B) 45 s
(C) 27 s (D) None of these
Ans. [C]
Effective speed of two trains = (80 – 40) = 40 km/hr. (Since they
are moving in the same direction as inferred from the word
‘overtakes’). At this speed in 54 s, they would travel an effective
distance of 403600
 54
= 0.6 km or 600 m.
This effective distance should be equal to the sum of the lengths
of the two trains. So, if length of the express train is L, length of
the goods train will be 2L. Hence, our equation will be L + 2L =
600
or L = 200 m.
So, the time taken by this train to cross a platform 400 m long
will be
200  400
  27 s
 5
 80 
18 

(Note that we have converted the denominator in metres per


second. Hence, the factor of 58)

40. P and Q are two positive integers such that PQ = 64. Which
of the following cannot be the value of P + Q?
(A) 20 (B) 65
(C) 16 (D) 35
Ans. [D]
If we were to express 64 as product of two positive integers, we
can get the following combinations: (64×1), (32×2), (16×4),
(8×8). Thus, we find that P+Q cannot be 35.

41. Lance, Sally, Joy and Fred are chosen for the team. In how
many ways can the three starters be chosen?
(A) 2 (B) 4
(C) 6 (D) 8
Ans. [B]
When three players start, one is the alternate. Because any of the
four players might be the alternate, there are four ways to select a
starting team: Lance-Sally-Joy, Lance-Sally-Fred, Lance-Joy-
Fred and Sally-Joy-Fred.

42. After Sally takes 20 shots, she has made 55% of her shots.
After she takes 5 more shots, she raises her percentage to 56%.
How many of the last 5 shots did she make?
(A) 1 (B) 2
(C) 3 (D) 4
Ans. [C]
If Sally makes 55% of her 20 shots, she makes 0.55×20 = 11
shots. If Sally makes 56% of her 25 shots, she makes 0.56×25 =
14 shots. So she makes 14–11 = 3 of the last 5 shots.

43. An athlete's target heart rate, in beats per minute, is 80%


of the theoretical maximum heart rate. The maximum heart
rate is found by subtracting the athlete's age, in years, from
220. To the nearest whole number, what is the target heart rate
of an athlete who is 26 years old?

(A) 134 (B) 155


(C) 176 (D) 194
Ans. [B]
A 26-year-old's target heart rate is 0:8(220–26) = 155.2 beats per
minute. The nearest whole number is 155.

44. The numbers -2, 4, 6, 9 and 12 are rearranged according to


these rules:
1. The largest isn't first, but it is in one of the first three
places.
2. The smallest isn't last, but it is in one of the last three
places.
3. The median isn't first or last.
What is the average of the first and last numbers?
(A) 3.5 (B) 5
(C) 6. (D) 7.5
Ans. [C]
The largest, smallest and median occupy the three middle places,
so the other two numbers, 9 and 4, are in the first and last places.
The average of 9 and 4 is 9 2 4  6.5

45. Ten years ago, the ages of the members of a joint family of
eight people added up to 231 years. Three years later, one
member died at the age of 60 years and a child was born
during the same year. After another three years, one more
member died, again at 60, and a child was born during the
same year. The current average age of this eight-member joint
family is nearest to
(A) 23 years (B) 22 years
(C) 21 years (D) 24 years
Ans. [D]
The total age of all the eight people in the family = 231 As per
the information given in the question, the total age of all the
people in the family=231+3×8–60+0=195 Similarly, the total
age of the people in the family four years ago = 195 + 3 × 8 –
60 + 0 = 159.
Therefore, the current average age of all the people in the
family
159  32
  24 years
8

46. What is the area enclosed by the geoboard quadrilateral


below?

(A) 15 (B) 18 12
1
(C) 22
2
(D) 27
Ans. [C]
47. Thirteen black and six white hexagonal tiles were used to
create the figure below. If a new figure is created by attaching
a border of white tiles with the same size and shape as the
others, what will be the difference between the total number
of white tiles and the total number of black tiles in the new
figure?

(A) 5 (B) 7
(C) 11 (D) 12
Ans. [C]

The next border requires an additional 6×3=18 white tiles. A


total of 24 white and 13 black tiles will be used, so the
difference is 24–13 = 11.

48. Three friends have a total of 6 identical pencils, and each one
has at least one pencil. In how many ways can this happen?
(A) 1 (B) 3
(C) 6 (D) 10
Ans. [D]
The largest number of pencils that any friend can have is four.
There are 3 ways that this can happen: (4; 1; 1), (1; 4; 1) and
(1; 1; 4). There are 6 ways one person can have 3 pencils: (3;
2; 1), (3; 1; 2), (2; 3; 1), (2; 1; 3), (1; 2; 3) and (1; 3; 2). There
is only one way all three can have two pencils each: (2; 2; 2).
The total number of possibilities is 3 + 6 + 1 = 10
49. Spinners A and B are spun. On each spinner, the arrow is
equally likely to land on each number. What is the probability
that the product of the two spinners' numbers is even?

1 1
(A) 4
(B) 3

1 2
(C) 2
(D) 3

Ans. [D]
In eight of the twelve outcomes the product is even: 1 × 2, 2
×1, 2 ×2, 2 × 3, 3 × 2, 4 × 1, 4 × 2, 4 × 3. In four of the twelve,
the product is odd: 1 × 1, 1 ×3, 3 ×1, 3 ×3. So the probability
8 2
that the product is even is 12 or .
3

50. At a party there are only single women and married men with
their wives. The probability that a randomly selected woman
is single is 23 . What fraction of the people in the room are
married men?
1 3
(A) 3
(B) 8

2 5
(C) 5
(D) 12

Ans. [B]
Because 25 of all the women in the room are single, there are
two single women for every three married women in the
room. There are also two single women for every three
married men in the room. So out of every 2 + 3 + 3 = 8 people,
3 are men. The fraction of the people who are married men is
3
8
.
 Thank You
for downloading the PDF

FREE LIVE ONLINE

MASTER CLASSES
FREE Webinars by Expert Teachers

FREE MASTER CLASS SERIES


 For Grades 6-12th targeting JEE, CBSE, ICSE & much more Register for FREE
 Free 60 Minutes Live Interactive classes everyday
 Learn from the Master Teachers - India’s best Limited Seats!

You might also like